i will give brainliest and 5 stars if you help ASAP​

I Will Give Brainliest And 5 Stars If You Help ASAP

Answers

Answer 1

Answer:

[tex] Area = 240 m^2 [/tex]

Step-by-step explanation:

The area of the right triangle above = [tex] \frac{1}{2}*base*height [/tex].

Where,

base = 16 m

height = 30 m

Plug in the above values into the area formula:

[tex] Area = \frac{1}{2}*16*30 [/tex]

[tex] Area = 8*30 [/tex]

[tex] Area = 240 m^2 [/tex]

Answer 2

Refer to the attachment for solution.

I Will Give Brainliest And 5 Stars If You Help ASAP

Related Questions

Find the distance between the points. Give an exact answer and an approximation to three decimal places.
(3.1,0.3) and (2.7. - 4.9)
The exact distance is
(Simplify your answer. Type an exact ans

Answers

Answer:  sqrt(27.2) =approx 5.215

Step-by-step explanation:

The distance between 2 points can be calculated using Phitagor theorem

L= sqrt( (x1-x2)²+(y1-y2)²)

Where x1, y1 are the coordinates of the first point and  x2, y2 are the coordinates of the 2-nd point.

L=sqrt((3.1-2.7)²+(0.3-(-4.9))²)= sqrt(0.4²+5.2²)=sqrt(27.2) - this is exact answer.

sqrt(27.2)=5.21536...=approx 5.215

Review the example argument and reasoning below. Identify the form (inductive or deductive) of reasoning and the type (example, analogy, causal correlation, syllogism, sign, or causal generalization) of reasoning Raul uses to justify his argument. Then, apply the three tests of argumentative reasoning (quantity, quality, & opposition) to test this argument.

Raul believes that if someone’s eyes shift to the left when they are responding to a question it is evidence that they are lying. While interviewing Michael, Raul notices Michael's eyes shifting to the left frequently when answering questions. Later, Raul tells a coworker that Michael was not hired because Raul believed Michael had lied about his previous experience during the interview.

Answers

Answer:

inductive - . Inductive reasoning makes broad generalizations from specific observations.

casual correlation

quality ( i think)

Step-by-step explanation:

Answer:

A

Step-by-step explanation:

i just did it

4. Two unbiased coins are tossed. Calculate the probability that
(a) Two heads are obtained.
(b) One head and one tail is obtained.

Answers

2 coins have four possible outcomes: {HH, HT, TH, TT}

(a) P(2 heads) = 1/4

(b) P(1 head and 1 tail) = 2/4 = 1/2

NO LINKS OR ANSWERING QUESTIONS YOU DON'T KNOW!!!

1. How can a matrix be used to solve a system of equations? Demonstrate by solving the following system. Show your work. In other words, use a problem of system of equations problem as an example.

Answers

Answer:

Step-by-step explanation:

Assuming the system is solvable in the first place, create an augmented matrix of coefficients from the equations. Then put the matrix into reduced row echelon form.

Example is attached.

"Demonstrate by solving the following system."

You need to provide the system of equations.

Decide all proper subsets of A { 8 ,7 ,6 ,5 ,4 ,3 ,2 ,1} = A 1- { 4 ,3 ,2 ,1} 2- { } 3- { 9 ,8 ,7 } 4- { 11 ,2} 5- { 5 }

Answers

Answer:

A, E

Step-by-step explanation:

There should be 2^8-1 proper subsets of A. Its every one besides { }

Complete the point-slope equation of the line through (2,3)(7,4). Use exact numbers. y-4=


Please help me, I would really appreciate it!

Answers

Answer:

The answer is

[tex]y - 4 = \frac{1}{5} (x - 7)[/tex]

Step-by-step explanation:

To find the equation of a line given two points first find the slope and use the formula

[tex] y - y_{1} = m(x - x_{1})[/tex]

Where m is the slope

To find the slope we use the formula

[tex]m = \frac{y2 - y1}{x2 - x1} [/tex]

The slope of the line using points

(2,3)(7,4) is

[tex]m = \frac{4 - 3}{7 - 2} = \frac{1}{5} [/tex]

Equation of the line using point (7,4) and slope 1/5 is

[tex]y - 4 = \frac{1}{5} (x - 7)[/tex]

Hope this helps you

Answer:

y-4=1/5(x-3)

Step-by-step explanation:

We plug in the x's and the y's and find the slope with:

[tex](y-y_{1} )/ x-x_{1})=m[/tex]

What is the circumference of the following circle?

Answers

Answer:

The answer is 157 in

Step-by-step explanation:

Circumference of a circle = 2πr

where

r is the radius

From the above question

radius = 25 in.

Substitute this value into the above formula

That's

Circumference = 2(25)π

= 50π

= 157.079

We have the final answer as

Circumference = 157 in

Hope this helps you

x+9=13352643-2x answer get brainliest

Answers

Answer:

4450878

Step-by-step explanation:

NEED HELP ASAP!!!!!!!!!!

Answers

Answer:

Hey there!

A is correct. The +2 means shifted up two units, 1/2 means compressed by a factor of 1/2, and the -3 means to the left of three units.

Let me know if this helps :)

What is the domain of f(x)=2/5x+6​

Answers

Answer:

Look at that picture

Step-by-step explanation:

Determine the convergence or divergence of the sequence with the given nth term. If the sequence converges, find its limit. (If the quantity diverges, enter DIVERGES.) an = 1/sqrt(n)

Answers

This sequence converges to 0.

Proof: Recall that

[tex]\displaystyle\lim_{n\to\infty}\frac1{\sqrt n}=0[/tex]

is to say that for any given [tex]\varepsilon>0[/tex], there is some [tex]N[/tex] for which [tex]\left|\frac1{\sqrt n}-0\right|=\frac1{\sqrt n}<\varepsilon[/tex] for all [tex]n>N[/tex].

Let [tex]N=\left\lceil\frac1{\varepsilon^2}\right\rceil[/tex]. Then

[tex]n>\left\lceil\dfrac1{\varepsilon^2}\right\rceil\ge\dfrac1{\varepsilon^2}[/tex]

[tex]\implies\dfrac1n<\varepsilon^2[/tex]

[tex]\implies\dfrac1{\sqrt n}<\varepsilon[/tex]

as required.

Please answer this correctly without making mistakes

Answers

Answer:

[tex]\large \boxed{\mathrm{4/5 \ cups}}[/tex]

Step-by-step explanation:

Subtract 1/10 from 9/10 to find out how much is left.

9/10 - 1/10

8/10 = 4/5

Answer:

4/5 cups

Step-by-step explanation:

[tex]Volume\:of \: syrup \:in \:cup\:from\:jug = \frac{9}{10}\\\\ She \:took\: \frac{1}{10} from \:the\:cup\:into\:the \:jug \\\\Volume \:of syrup\:in\:cup=?\\\\\frac{9}{10} -\frac{1}{10} \\\\= \frac{4}{5} cups[/tex]

Consider the quadratic equation 4x2 - 11x - 3 = 0. Which of the following shows
this equation rewritten and ready to solve using factoring by grouping?
A) 4x2 - 9x - 2x - 3 = 0
B) 4x2 - 6x - 5x - 3 = 0
C) 4x2 - 12x + x - 3 = 0
D) 4x2 + 12x - x-3 = 0

Answers

Answer:

C

Step-by-step explanation:

The required equation that is rewritten and ready to solve is 4x² - 12x + x - 3 = 0. Option C is correct.

Given that,
The quadratic equation 4x² - 11x - 3 = 0 The equations is rewritten and ready to solve using factoring by grouping is to be determined.

What is simplification?

The process in mathematics to operate and interpret the function to make the function or expression simple or more understandable is called simplifying and the process is called simplification.

Here,
4x² - 11x - 3 = 0
4x² - 12x + x - 3 = 0
4x[x - 3] + 1[x - 3] = 0
[4x + 1][x - 3] = 0


Thus, the required equation that is rewritten and ready to solve is 4x² - 12x + x - 3 = 0. Option C is correct.

Learn more about simplification here:

https://brainly.com/question/12501526

#SPJ2

Let REPEAT TM = { | M is a TM, and for all s ∈ L(M), s = uv where u = v }. Show that REPEATTM is undecidable. Do not use Rice’s Theorem.

Answers

Answer:

Step-by-step explanation:

Let REPEAT [tex]_{TM[/tex]= { | M is a TM, and for all s ∈ L(M), s = uv where u = v }

To prove that REPEAT [tex]_{TM[/tex]  is undecidable.

Let  REPEAT [tex]_{TM[/tex] {| M is a TM that does not accept M}

Then, we form a TM  u for L by applying TM v as a subroutine.

Assume Repeat   is decidable

Let M be the algorithm that  TM which decides the REPEATU = on input "s" simulate the M

Accept; if M ever enters the accept state

Reject; if M ever enters the reject state

U does not decide the REPEAT as it may loop over s

so REPEAT is undecidable

Question 2b only! Evaluate using the definition of the definite integral(that means using the limit of a Riemann sum

Answers

Answer:

Hello,

Step-by-step explanation:

We divide the interval [a b] in n equal parts.

[tex]\Delta x=\dfrac{b-a}{n} \\\\x_i=a+\Delta x *i \ for\ i=1\ to\ n\\\\y_i=x_i^2=(a+\Delta x *i)^2=a^2+(\Delta x *i)^2+2*a*\Delta x *i\\\\\\Area\ of\ i^{th} \ rectangle=R(x_i)=\Delta x * y_i\\[/tex]

[tex]\displaystyle \sum_{i=1}^{n} R(x_i)=\dfrac{b-a}{n}*\sum_{i=1}^{n}\ (a^2 +(\dfrac{b-a}{n})^2*i^2+2*a*\dfrac{b-a}{n}*i)\\[/tex]

[tex]=(b-a)^2*a^2+(\dfrac{b-a}{n})^3*\dfrac{n(n+1)(2n+1)}{6} +2*a*(\dfrac{b-a}{n})^2*\dfrac{n (n+1)} {2} \\\\\displaystyle \int\limits^a_b {x^2} \, dx = \lim_{n \to \infty} \sum_{i=1}^{n} R(x_i)\\\\=(b-a)*a^2+\dfrac{(b-a)^3 }{3} +a(b-a)^2\\\\=a^2b-a^3+\dfrac{1}{3} (b^3-3ab^2+3a^2b-a^3)+a^3+ab^2-2a^2b\\\\=\dfrac{b^3}{3}-ab^2+ab^2+a^2b+a^2b-2a^2b-\dfrac{a^3}{3} \\\\\\\boxed{\int\limits^a_b {x^2} \, dx =\dfrac{b^3}{3} -\dfrac{a^3}{3}}\\[/tex]

You have $2,000 on a credit card that charges a 16% interest rate. If you want to pay off the credit card in 5 years, how much will you need to pay each month (assuming you don't charge anything new to the card)?

Answers

9514 1404 393

Answer:

  $48.64

Step-by-step explanation:

The monthly payment amount is given by the amortization formula ...

  A = P(r/n)/(1 -(1 +r/n)^(-nt))

where P is the loan amount, r is the annual interest rate compounded n times per year for t years.

Here, you have P=2000, r=0.16, n=12 (months per year), t=5 (years), so the payment is ...

  A = $2000(0.16/12)/(1 -(1 +0.16/12)^(-12·5)) = $320/(12(0.54828942))

  A ≈ $48.636 ≈ $48.64

You will need to pay $48.64 each month to pay off the charge in 5 years.

PLEASE HELP IM SO LOST

1. Ted is working on his financial plan and lists all of his income and expenses in the spreadsheet below.
А
B
Net Pay
$5,000
2
Interest on Deposits $0
3 Income from Investments $225
4 Rent
$3,000
5 Utilities
$250
6 Satellite Dish
$175
7 Cell Phone Plan
$135
8 Car Payment
$385
9 Groceries
$200
10 Insurance
$380
11 Recreation
$400
What is Ted's net cash flow?
2. Tamara earns $8 an hour at her job working 25 hours per week. If her net pay is 78% of her paycheck
and she has no other sources of income, what is Tamara's monthly cash inflow? (Assume there are 4
pays per month.)

Answers

Answer:  1) $300     2) $624

Step-by-step explanation:

[tex]\begin{array}{l||l|l}\underline{\quad \text{Item}\qquad \qquad \qquad \qquad}&\underline{\text{Income} }&\underline{\text{Expense}}\\\text{Net Pay}&5000&\\\text{Interest on Deposits}&0&\\\text{Income from Investments}&225&\\\text{Rent}&&3000\\\text{Utilities}&&250\\\text{Satellite Dish}&&175\\\text{Cell Phone Plan}&&135\\\text{Car Payment}&&385\\\text{Groceries}&&200\\\text{Insurance}&&380\\\underline{\text{Recreation}\qquad \qquad \qquad}&\underline{\qquad \quad }&\underline{400\qquad}\\\end{array}[/tex]

TOTALS                              5225      4925

Net Cash Flow = Income - Expenses

                        = 5225 - 4925

                        = 300

*************************************************************************************

[tex]\dfrac{25\ hours}{week}\times \dfrac{\$8}{hour}\times 4\ weeks\times 78\%\\\\\\=25\times \$8 \times 0.78\\\\= \$624[/tex]

7.619 by 10^-3



7.254 by 10^2​

Answers

Answer:

0.007619

0.07254

Step-by-step explanation:

1)7.619*10^-3

0.007619

2)7.254*10^2

0.07254

Explanation:

7.619*10^-3

The number here is 7.619 and the number written in scientific notation has minus 3 as its exponent.

.007.619

So the distance between the first decimal point and the second decimal is only three numbers.

Since it is exponent is minus three.

Another way to get the answer.

[tex]7.619 \times 10 {}^{ - 3} = \frac{7619}{1000} \times \frac{1}{1000} = \frac{7619}{1000000} = 0.007619 [/tex]

This applies to the second one too.

Hope this helps ;) ❤❤❤

If I get paid $100 a week but my car cost $4, 990 how long will I have to save to officially buy my car?

Answers

50 weeks 49 weeks to pay $100 and the 50th week you pay $90

Answer:

50 weeks.

Step-by-step explanation:

Did the math like u should've

Calculate how many different sequences can be formed that use the letters of the given word. Leave your answer as a product of terms of the form C(n, r). HINT [Decide where, for example, all the s's will go, rather than what will go in each position.]
georgianna
A) C(10, 7)
B) C(2, 10)C(1, 8)C(1, 7)C(1, 6)C(1, 5)C(2, 4)C(2, 2)
C) C(10, 2)C(8, 1)C(7, 1)C(6, 1)C(5, 1)C(4, 1)C(3, 1)C(2, 1)C(1, 1)
D) 10 · C(10, 2)C(8, 1)C(7, 1)C(6, 1)C(5, 1)C(4, 2)C(2, 2)
E) C(10, 2)C(8, 1)C(7, 1)C(6, 1)C(5, 1)C(4, 2)C(2, 2)

Answers

Answer: E) C(10, 2)C(8, 1)C(7, 1)C(6, 1)C(5, 1)C(4, 2)C(2, 2)

Step-by-step explanation:

According to the combinations: Number of ways to choose r things out of n things = C(n,r)

Given word: "georgianna"

It is a sequence of 10 letters with 2 a's , 2 g's , 2 n's , and one of each e, o,r, i.

If we think 10 blank spaces, then in a sequence we need 2 spaces for each of g.

Number of ways = C(10,2)

Similarly,

1 space for 'e' → C(8,1)

1 space for 'o' → C(7,1)

1 space for 'r' → C(6,1)

1 space for 'i' → C(5,1)

1 space for 'a' → C(4,2)

1 space for 'n' → C(2,2)

Required number of different sequences  = C(10,2) ×C(8,1)× C(7,1)× C(6,1)×C(5,1)×C(2,2).

Hence, the correct option is E) C(10, 2)C(8, 1)C(7, 1)C(6, 1)C(5, 1)C(4, 2)C(2, 2)

Researchers recorded that a certain bacteria population declined from 120,000 to 200 in 36 hours. At this rate of decay, how many bacteria will there be in 31 hours? Round to the nearest whole number.

Answers

Answer: There will 486 bacteria in 31 hours.

Step-by-step explanation:

The population decay in bacteria is exponential.

Exponential function : [tex]y=Ab^x[/tex], where A = initial population, b multiplication decay factor, t= time:

As per given:

Initial population: [tex]A=120,000[/tex]

After 36 hours, population = [tex]120000(b^{36})=200[/tex]

Divide both sides by 120,000 we get

[tex]b^{36}= 0.00167[/tex]

Taking natural log on both sides , we get

[tex]36\ln b=\ln 0.00167\\\\\Rightarrow\ b=e^{\left(\frac{\ln0.00167}{36}\right)}=0.83724629\approx0.8372[/tex]

At x= 31,

[tex]y=120000(0.8372)^{31}=120000\times0.00405234\approx486[/tex]

Hence, there will 486 bacteria in 31 hours.

convert 407 in base 8 to decimal​

Answers

[tex]4\cdot8^2+0\cdot8^1+7\cdot8^0=256+7=263[/tex]

[tex]407_8=263_{10}[/tex]

A professional soccer player kicked a ball across the field. The ball’s height, in meters, is modeled by the function graphed below. What's the average rate of change between the point when the ball reached its maximum height and the point where it hit the ground?

Answers

Answer:

Hey there!

You can think of the rate of change as the slope of a quadratic function- here we see that it is 9/-3, or - 3.

Let  me know if this helps :)

Answer:

–3 meters per second

Step-by-step explanation:

A car dealer's markup on every car they sell is 20%. For what price did the dealership buy a car that they sold for $18,600?

Answers

Answer:

buying price = $15,500

Step-by-step explanation:

selling price 20% more than the buying price

let the buying price be 100% then;

selling price = 120%

120% = $18,600

100% = ?

(100 × 18600) ÷ 120

= $15,500

The solutions to the equation $(x+1)(x+2) = x+3$ can be written in the form $m+\sqrt n$ and $m-\sqrt n$, where $m$ and $n$ are integers. What is $m+n$?

Answers

Answer:

1

Step-by-step explanation:

Hello, please consider the following.

First, we develop and move everything to the left side, then we solve the equation, using the discriminant.

Finally, we get the expression of the two solutions and we can conclude.

[tex](x+1)(x+2)=(x+3)\\\\<=> x^2+3x+2-x-3=0\\\\<=>x^2+2x-1=0\\\\\Delta=b^2-4ac=4+4=8\\\\x_1=\dfrac{-2-\sqrt{8}}{2}=\dfrac{-2-2\sqrt{2}}{2}=-1-\sqrt{2}\\\\x_2=\dfrac{-2+\sqrt{8}}{2}=-1+\sqrt{2}[/tex]

So, m=-1 and n = 2

m+n = -1 + 2 = 1

Thank you

The value of m + n is 1.

What is an equation?

An equation is a mathematical statement that is made up of two expressions connected by an equal sign.

Example:

2x + 4 - 9 is an equation.

We have,

(x + 1)(x + 2) = x + 3

x² + 2x + x + 2 = x + 3

x² + 3x + 2 - x - 3 = 0

x² + 2x - 1 = 0

This is in the form of ax² + bx + c = 0

a = 1, b = 2, and c = -1

Now,

Using the determinant.

x = -b ± √(b² - 4ac) / 2a

x = -2 ± √(4 + 4) / 2

x = (-2 ± 2√2) / 2

x = (-1 ± √2)

x = -1 + √2

x = -1 - √2

Now,

The solutions can be written in the form of (m + √n) and (m - √n).

This means,

m + √n = -1 + √2

m - √n = -1 - √2

m = -1 and n = 2

Now,

m + n

= -1 + 2

= 1

Thus,

m + n is 1.

Learn more about equations here:

https://brainly.com/question/17194269

#SPJ2

if sin150=1/2 then find sin75

Answers

Sin75° = Sin(30° + 45°)

= Sin30°.Cos45° + Sin45°.Cos30°

= 1/2 . √2/2 + √2/2.√3/2

= √2/2 ( 1/2 + √3/2 )

= √2/2 ((1+√3 ) /2 )

= (√2 + √6 )/ 4

How many solutions does the following equation have ?
−3x+9−2x=−12−5x

Answers

[tex]\text{Solve for x:}\\\\-3x+9-2x=-12-5x\\\\\text{Combine like terms}\\\\-5x+9=-12-5x\\\\\text{Add 5x to both sides}\\\\9=-12\\\\\text{Since that's not valid, there would be no solutions}\\\\\boxed{\text{No solutions}}[/tex]

What is the mulitplicative rate of change for the exponential function f(x) = 2 (5over2) to the negative x power ?

Answers

Answer:

  2/5

Step-by-step explanation:

  f(x) = 2(5/2)^-x = 2(2/5)^x

The multiplicative rate of change is the base of the positive exponent, 2/5.

write a thirdthird-degree polynomial expression that has only two terms with a leading term that has a coefficient of five and a constant of negative two ​

Answers

Answer:

5x^3-2

[tex]ax^{3} +bx^{2} +cx+d\\5x^{3}-given\\ d=-2-given\\5x^{3} -2[/tex]

Answer: [tex]5x^3 - 2[/tex]

Explanation:

The two terms are [tex]5x^3[/tex] and [tex]2[/tex]. Terms are separated by either a plus or minus.

We can write it as [tex]5x^3+(-2)[/tex] which is an equivalent form. Here the two terms are [tex]5x^3[/tex] and [tex]-2[/tex]. This is because adding a negative is the same as subtracting.

The coefficient is the number to the left of the variable.

The degree is the largest exponent, which helps form the leading term.

The third degree polynomial written above is considered a cubic binomial. "Cubic"  refers to the third degree, while "binomial" means there are 2 terms.

We can write something like [tex]5x^3[/tex] as 5x^3 when it comes to computer settings.

A 95% confidence interval for the mean number of television per American household is (1.15, 4.20). For each of the following statements about the above confidence interval, choose true or false.
a. The probability that u is between 1.15 and 4.20 is .95.
b. We are 95% confident that the true mean number of televisions per American household is between 1.15 and 4.20.
c. 95% of all samples should have x-bars between 1.15 and 4.20 televisions.
d. 95% of all American households have between 1.15 and 4.20 televisions
e. Of 100 intervals calculated the same way (95%), we expect 95 of them to capture the population mean.
f. Of 100 intervals calculated the same way (95%), we expect 100 of them to capture the sample mean.

Answers

Answer:

a. False

b. True

c. False

d. False

e.True

f. True

Step-by-step explanation:

The 95% is confidence interval its not a probability estimate. The probability will be different from the confidence interval. Confidence interval is about the population mean and is not calculated based on sample mean. Every confidence interval contains the sample mean. There is 95% confidence that the number of televisions per American household is between 1.15 to 4.20.

Other Questions
EFT, Inc. EFT, Inc. wants to empower and engage its employees. They have several teams consisting of highly skilled employees and no one person on these teams has a specifically assigned position. Instead, employees lead each other as the occasion arises so there is no formal hierarchy or organizational chart. This type of leadership is most effective when: Follow the process of completing the square to solve x2 = -4x + 3 find the common ratio 3, 9, 27, 81 I need help on my new iPhone 7 that I got I transfer my Sim card from my Samsung But before And after I put my Sim card in it was saying this please help...and yeah:( Ill let give you fifty .. Which sentence from the passage includes a coordinatingconjunction?A "He noted that private schools, which required students towear standard uniforms, benefitted from such regulations."(paragraph 2)B However, I feel that Mr. Crockett's editorial did not present acomplete picture of the issue, and I would like your readers tosee the gaps in his arguments. (paragraph 1)C Furthermore, comparing public to private schools iscomparing apples to oranges." (paragraph 3)0D He wrote that wearing school uniforms might result instudents feeling more connected to their peers and the schoolcommunity, and foster a stronger sense of identity andfellowship. (paragraph 7) At the beginning of the day the stock market goes up 60 1/2 points and stays at this level for most of the day. At the end of the day the stock market goes down 100 1/4 points from the high at the beginning of the day. What is the total change in the stock market from the beginning of the day to the end of the day? let d equal the distance in meters and t equal the time in seconds. Which is a direct variation equation for this relationship Item 3Item 3 Cutter Enterprises purchased equipment for $87,000 on January 1, 2018. The equipment is expected to have a five-year life and a residual value of $4,800. Using the sum-of-the-years'-digits method, depreciation for 2018 and book value at December 31, 2018, would be I need answering ASAP please and thank you Two years ago the population of a town was 40000. The population of the town at present has reached 44100. Calculate the population growth rate of the city. what is crop rotations =X square there is written at the end The total cost for a bucket of popcorn and 4 movie tickets is $56. The total cost for the same size bucket of popcorn and 6 movie tickets is $80. The cost of a bucket of popcorn is $8. Which equation represents the relationship between y, the total cost of the popcorn and movie tickets, and x, the number of movie tickets that are purchased? If 120 is divided into 3 parts which are proportional to 1, [tex]\frac{1}{2} [/tex], and [tex]\frac{1}{6}[/tex], what is the middle part? Plsss help with this ASAP TT 3Select the correct answer.The angle of depression between the top of a 100-foot cliff and a ship approaching the shore is 37.cliff top37100feet37shipdWhat is the approximate distance, d, between the bottom of the cliff and the ship?166.2 feetOB. 60.2 feet.75.4 feetOD.132.7 feetResetNext A sinusoidal voltage is displayed on an oscilloscope screen. The separation on the screen between two adjacent peaks is 5.5 divisions, and you notice that the sweep speed is set to 1 ms per division. What is the frequency of the sinusoidal voltage A bicycle has wheels that are 60 cm in diameter. What is the angular speed of these wheels when it is moving at 4.0 m/s A family that includes a mixture of parents and children unrelated by blood isalso described as a(n): A. Biological family.B. Multigenerational family C. Blended family D. Extended family Translate the following phrase into an algebraic expression using the variable m. Do not simplify,the cost of renting a car for one day and driving m miles if the rate is $39 per day plus 45 cents per mile